上一主题:CMA 2010新考试官方版联系题part1+part2+essay (英语版)
下一主题:[下载]2008淘汰的真題 2008 Retired CMA Exam Questions
返回列表 发帖

 [1] Gleim #: 1.1.1 -- Source: Publisher
Below are the grocers’ demand schedules for Palm Valley Grapes. Assuming that John, Towny, and Dorothea are the only
three customers of Palm Valley Grapes, which of the following sets of prices and output levels will be on the market
demand curve?
Price John Towny Dorothea
of Grapes Qdx Qdx Qdx
$6 0 1 0
5 1 2 0
4 2 4 0
3 3 6 1
2 4 8 2
1 5 9 3
A. ($6, 2); ($1, 17)
B. ($5, 3); ($1, 17)
C. ($4, 4); ($2, 12)
D. ($4, 0); ($1, 9)
Answer (A) is incorrect because demand would be 1 when the price is $6.
Answer (B) is correct. This type of problem is solved by means of trial and error. Check each of the answer
alternatives to determine whether both points represent a level of demand for a given price. Answer (B) is correct
because total demand would be 3 at a price of $5, and total demand would be 17 (5 + 9 + 3) at a price of $1.
Answer (C) is incorrect because demand would be 6, not 4, when the price is $4, and demand would be 14, not 12,
when the price is $2.
Answer (D) is incorrect because demand would be 6 when the price is $4 and 17 when the price is $1.
[2] Gleim #: 1.1.2 -- Source: CMA 1293 1-10
If a group of consumers decide to boycott a particular product, the expected result would be
A. An increase in the product price to make up lost revenue.
B. A decrease in the demand for the product.
C. An increase in product supply because of increased availability.
D. That demand for the product would become completely inelastic.
Answer (A) is incorrect because reduced demand will drive the price downward.
Answer (B) is correct. A consumer boycott will decrease the demand for a product (shift the demand curve to the
left). This decrease in demand should lead to a lower price for the product assuming that supply is constant (the
supply curve does not shift).
Answer (C) is incorrect because supply remains unchanged in the short run.
Answer (D) is incorrect because, if demand is inelastic, customers will continue buying the product regardless of
the price; a boycott, however, means that consumers will stop buying the product.

AJAW8HL3.rar (389.24 KB)

**************************** CMA学习记录 http://hi.baidu.com/gowithme ****************************

what is this。[em03]

TOP

 啥东东呢

TOP

什么东西啊?

TOP

看看再说~~

TOP

看看再说

TOP

Thanks!不過好貴喔

TOP

啥东西??

TOP

so expenses, thanks

TOP

谢谢

TOP

返回列表
上一主题:CMA 2010新考试官方版联系题part1+part2+essay (英语版)
下一主题:[下载]2008淘汰的真題 2008 Retired CMA Exam Questions